Does the improper integral $int_0^infty e^{-x^2}dx$ converge?












7












$begingroup$


I want to show the convergence of the following improper integral $int_0^infty e^{-x^2}dx$.
I try to use comparison test for integrals
$x≥0$, $-x ≥0$, $-x^2≥0$ then $e^{-x^2}≤1$. So am ending with the fact that $int_0^infty e^{-x^2}dx$ converges if $int_0^infty dx$ converges but I don’t appreciate this. Thanks










share|cite|improve this question











$endgroup$












  • $begingroup$
    Please use LaTeX. Is this integral of $int_0^{infty}e^{-x^2},dx$ (int_0^{infty}e^{-x^2}dx), $int_0^{infty}(e-x^2),dx$ (int_0^{infty}(e-x^2)dx$, or what?
    $endgroup$
    – Arturo Magidin
    Jan 6 '12 at 4:32








  • 6




    $begingroup$
    I assume you mean $int_0^infty e^{-x^2} dx$. Hint: compare to $int_0^infty e^{-x} dx$.
    $endgroup$
    – Robert Israel
    Jan 6 '12 at 4:32






  • 1




    $begingroup$
    @neemy: Yes the integral $int_{0}^{infty} e^{-x^2} dx$ converges and it's value is $sqrt{pi}$
    $endgroup$
    – user9413
    Jan 6 '12 at 4:34






  • 2




    $begingroup$
    There is a wonderful math markup language called LaTeX which this site supports and allows you to communicate math much more clearly and legibly. Learn it; it will help you here and elsewhere. I have edited your question with proper formatting. If the new version is not the question you were trying to ask, just let me know.
    $endgroup$
    – Alex Becker
    Jan 6 '12 at 4:37






  • 2




    $begingroup$
    @Chandrasekhar You probably meant to write the value as $sqrt{pi}/2$, not $sqrt{pi}$.
    $endgroup$
    – Dilip Sarwate
    Jan 6 '12 at 13:32
















7












$begingroup$


I want to show the convergence of the following improper integral $int_0^infty e^{-x^2}dx$.
I try to use comparison test for integrals
$x≥0$, $-x ≥0$, $-x^2≥0$ then $e^{-x^2}≤1$. So am ending with the fact that $int_0^infty e^{-x^2}dx$ converges if $int_0^infty dx$ converges but I don’t appreciate this. Thanks










share|cite|improve this question











$endgroup$












  • $begingroup$
    Please use LaTeX. Is this integral of $int_0^{infty}e^{-x^2},dx$ (int_0^{infty}e^{-x^2}dx), $int_0^{infty}(e-x^2),dx$ (int_0^{infty}(e-x^2)dx$, or what?
    $endgroup$
    – Arturo Magidin
    Jan 6 '12 at 4:32








  • 6




    $begingroup$
    I assume you mean $int_0^infty e^{-x^2} dx$. Hint: compare to $int_0^infty e^{-x} dx$.
    $endgroup$
    – Robert Israel
    Jan 6 '12 at 4:32






  • 1




    $begingroup$
    @neemy: Yes the integral $int_{0}^{infty} e^{-x^2} dx$ converges and it's value is $sqrt{pi}$
    $endgroup$
    – user9413
    Jan 6 '12 at 4:34






  • 2




    $begingroup$
    There is a wonderful math markup language called LaTeX which this site supports and allows you to communicate math much more clearly and legibly. Learn it; it will help you here and elsewhere. I have edited your question with proper formatting. If the new version is not the question you were trying to ask, just let me know.
    $endgroup$
    – Alex Becker
    Jan 6 '12 at 4:37






  • 2




    $begingroup$
    @Chandrasekhar You probably meant to write the value as $sqrt{pi}/2$, not $sqrt{pi}$.
    $endgroup$
    – Dilip Sarwate
    Jan 6 '12 at 13:32














7












7








7


2



$begingroup$


I want to show the convergence of the following improper integral $int_0^infty e^{-x^2}dx$.
I try to use comparison test for integrals
$x≥0$, $-x ≥0$, $-x^2≥0$ then $e^{-x^2}≤1$. So am ending with the fact that $int_0^infty e^{-x^2}dx$ converges if $int_0^infty dx$ converges but I don’t appreciate this. Thanks










share|cite|improve this question











$endgroup$




I want to show the convergence of the following improper integral $int_0^infty e^{-x^2}dx$.
I try to use comparison test for integrals
$x≥0$, $-x ≥0$, $-x^2≥0$ then $e^{-x^2}≤1$. So am ending with the fact that $int_0^infty e^{-x^2}dx$ converges if $int_0^infty dx$ converges but I don’t appreciate this. Thanks







real-analysis integration






share|cite|improve this question















share|cite|improve this question













share|cite|improve this question




share|cite|improve this question








edited Jan 6 '12 at 4:34









Alex Becker

49.2k6100161




49.2k6100161










asked Jan 6 '12 at 4:27









neemyneemy

112127




112127












  • $begingroup$
    Please use LaTeX. Is this integral of $int_0^{infty}e^{-x^2},dx$ (int_0^{infty}e^{-x^2}dx), $int_0^{infty}(e-x^2),dx$ (int_0^{infty}(e-x^2)dx$, or what?
    $endgroup$
    – Arturo Magidin
    Jan 6 '12 at 4:32








  • 6




    $begingroup$
    I assume you mean $int_0^infty e^{-x^2} dx$. Hint: compare to $int_0^infty e^{-x} dx$.
    $endgroup$
    – Robert Israel
    Jan 6 '12 at 4:32






  • 1




    $begingroup$
    @neemy: Yes the integral $int_{0}^{infty} e^{-x^2} dx$ converges and it's value is $sqrt{pi}$
    $endgroup$
    – user9413
    Jan 6 '12 at 4:34






  • 2




    $begingroup$
    There is a wonderful math markup language called LaTeX which this site supports and allows you to communicate math much more clearly and legibly. Learn it; it will help you here and elsewhere. I have edited your question with proper formatting. If the new version is not the question you were trying to ask, just let me know.
    $endgroup$
    – Alex Becker
    Jan 6 '12 at 4:37






  • 2




    $begingroup$
    @Chandrasekhar You probably meant to write the value as $sqrt{pi}/2$, not $sqrt{pi}$.
    $endgroup$
    – Dilip Sarwate
    Jan 6 '12 at 13:32


















  • $begingroup$
    Please use LaTeX. Is this integral of $int_0^{infty}e^{-x^2},dx$ (int_0^{infty}e^{-x^2}dx), $int_0^{infty}(e-x^2),dx$ (int_0^{infty}(e-x^2)dx$, or what?
    $endgroup$
    – Arturo Magidin
    Jan 6 '12 at 4:32








  • 6




    $begingroup$
    I assume you mean $int_0^infty e^{-x^2} dx$. Hint: compare to $int_0^infty e^{-x} dx$.
    $endgroup$
    – Robert Israel
    Jan 6 '12 at 4:32






  • 1




    $begingroup$
    @neemy: Yes the integral $int_{0}^{infty} e^{-x^2} dx$ converges and it's value is $sqrt{pi}$
    $endgroup$
    – user9413
    Jan 6 '12 at 4:34






  • 2




    $begingroup$
    There is a wonderful math markup language called LaTeX which this site supports and allows you to communicate math much more clearly and legibly. Learn it; it will help you here and elsewhere. I have edited your question with proper formatting. If the new version is not the question you were trying to ask, just let me know.
    $endgroup$
    – Alex Becker
    Jan 6 '12 at 4:37






  • 2




    $begingroup$
    @Chandrasekhar You probably meant to write the value as $sqrt{pi}/2$, not $sqrt{pi}$.
    $endgroup$
    – Dilip Sarwate
    Jan 6 '12 at 13:32
















$begingroup$
Please use LaTeX. Is this integral of $int_0^{infty}e^{-x^2},dx$ (int_0^{infty}e^{-x^2}dx), $int_0^{infty}(e-x^2),dx$ (int_0^{infty}(e-x^2)dx$, or what?
$endgroup$
– Arturo Magidin
Jan 6 '12 at 4:32






$begingroup$
Please use LaTeX. Is this integral of $int_0^{infty}e^{-x^2},dx$ (int_0^{infty}e^{-x^2}dx), $int_0^{infty}(e-x^2),dx$ (int_0^{infty}(e-x^2)dx$, or what?
$endgroup$
– Arturo Magidin
Jan 6 '12 at 4:32






6




6




$begingroup$
I assume you mean $int_0^infty e^{-x^2} dx$. Hint: compare to $int_0^infty e^{-x} dx$.
$endgroup$
– Robert Israel
Jan 6 '12 at 4:32




$begingroup$
I assume you mean $int_0^infty e^{-x^2} dx$. Hint: compare to $int_0^infty e^{-x} dx$.
$endgroup$
– Robert Israel
Jan 6 '12 at 4:32




1




1




$begingroup$
@neemy: Yes the integral $int_{0}^{infty} e^{-x^2} dx$ converges and it's value is $sqrt{pi}$
$endgroup$
– user9413
Jan 6 '12 at 4:34




$begingroup$
@neemy: Yes the integral $int_{0}^{infty} e^{-x^2} dx$ converges and it's value is $sqrt{pi}$
$endgroup$
– user9413
Jan 6 '12 at 4:34




2




2




$begingroup$
There is a wonderful math markup language called LaTeX which this site supports and allows you to communicate math much more clearly and legibly. Learn it; it will help you here and elsewhere. I have edited your question with proper formatting. If the new version is not the question you were trying to ask, just let me know.
$endgroup$
– Alex Becker
Jan 6 '12 at 4:37




$begingroup$
There is a wonderful math markup language called LaTeX which this site supports and allows you to communicate math much more clearly and legibly. Learn it; it will help you here and elsewhere. I have edited your question with proper formatting. If the new version is not the question you were trying to ask, just let me know.
$endgroup$
– Alex Becker
Jan 6 '12 at 4:37




2




2




$begingroup$
@Chandrasekhar You probably meant to write the value as $sqrt{pi}/2$, not $sqrt{pi}$.
$endgroup$
– Dilip Sarwate
Jan 6 '12 at 13:32




$begingroup$
@Chandrasekhar You probably meant to write the value as $sqrt{pi}/2$, not $sqrt{pi}$.
$endgroup$
– Dilip Sarwate
Jan 6 '12 at 13:32










6 Answers
6






active

oldest

votes


















17












$begingroup$

Write



$$int_0^infty e^{-x^2} , dx = int_0^1 e^{-x^2} , dx + int_1^infty e^{-x^2} , dx$$



The first integral on the right hand side is of a bounded function over a bounded interval, hence finite. For the second notice that we have $x^2 > x$ when $x > 1$. Therefore, $e^{-x^2} < e^{-x}$ for $x > 1$ and it follows that



$$int_1^infty e^{-x^2} , dx < int_1^infty e^{-x} , dx$$
$$= lim_{x to infty} -e^{-x} + e^{-1} = 1/e < infty$$






share|cite|improve this answer











$endgroup$





















    4












    $begingroup$

    I'm assuming you're asking about the convergence of $int_0^infty e^{-x^2}dx$. The easiest way that I can think of to prove this is to note that $e^{-x^2}$ is continuous and bounded, and hence integrable, on the interval $[0,1]$, and that on the remaining unbounded interval $[1,infty)$ it is a function everywhere bounded in absolute value by a function $e^{-x}$ that is integrable on that interval (seeing that $int_1^infty e^{-x}dx$ converges is a simple calculation, since $e^{-x}$ has an easy antiderivative). Thus, by the comparison test, $int_0^infty e^{-x^2}dx$ converges. I think this is the idea you're referring to in your question.



    $int_0^infty e^{-x^2}dx$ does not exist if $int_0^infty dx$ does, since the latter one most certainly does not converge (it equals $lim_{x to infty} x = infty$). While it is true that $e^{-x^2} le 1$ on $[0,infty)$, this fact isn't really helpful, because the comparison test only gives us information when the bounding function is itself integrable (otherwise you get absurdities like claiming that $int_0^infty e^{-x}dx$ diverges because $e^{-x} le x $ and $int_0^infty x dx$ diverges.)






    share|cite|improve this answer









    $endgroup$





















      3












      $begingroup$

      It does, and you can also compute its value:



      $int_{[0, infty) times[0, infty)} e^{-(x^2+y^2)} dx dy = int_{[0, infty)}big(int_{[0, infty)} e^{-(x^2+y^2)} dy big)dx = int_{[0, infty)} e ^ {-x^2}big(int_{[0, infty)} e^{-y^2} dy big)dx =$
      $= int_{[0, infty)} e ^ {-x^2}dx int_{[0, infty)} e^{-y^2} dy = big( int_{[0, infty)} e ^ {-x^2}dx big)^2$



      Then we can use polar coordinates:



      $int_{[0, infty) times[0, infty)} e^{-(x^2+y^2)} dx dy = int_0^{infty} big(int_0^{fracpi2}re^{-r^2}dtheta)dr = fracpi2int_0^{infty}re^{-r^2}dr = fracpi2 frac12 = fracpi4$



      Therefore:



      $int_{[0, infty)} e ^ {-x^2}dx = big(fracpi4big)^{frac12}$






      share|cite|improve this answer











      $endgroup$





















        1












        $begingroup$

        Exercise 3-41 of Spivak's "Calculus on Manifolds" outlines how to prove that $int_{-infty}^{infty}e^{-x^{2}}dx=sqrt{pi}$ using polar coordinates, a related double integral, and two useful families of regions over which to evaluate the double integral. The description is actually quite long. Ironically, he ends the problem with a quote attributed to Lord Kelvin (William Thomson), who was trying to convey the meaning of "mathematician" to a class: "A mathematician is one to whom "that" (i.e., $int_{-infty}^{infty}e^{-x^{2}}dx=sqrt{pi}$) is as obvious as that twice two makes four is to you." A bit harsh, I daresay, on Lord Kelvin's part. The reality being conveyed by Spivak is that it's obvious once you've done the necessary work.






        share|cite|improve this answer









        $endgroup$









        • 2




          $begingroup$
          Does Spivak quote the next part? "Liouville was a mathematician." I always thought that was a nice thing to say about someone.
          $endgroup$
          – Dylan Moreland
          Jan 6 '12 at 6:54










        • $begingroup$
          @Dylan-Yes. Spivak includes that statement.
          $endgroup$
          – Chris Leary
          Jan 6 '12 at 18:15



















        0












        $begingroup$

        Let $$f(x) = frac{1}{{e^x}^{2}}$$
        Now let $$g(x) = frac{1}{x^2 + 1}$$
        $$lim _{xrightarrow infty} = 0 $$ and $g(x)$ is a convergent integral, so $f(x)$ is convergent too.






        share|cite|improve this answer











        $endgroup$





















          -3












          $begingroup$

          A simple answer as @neemy… Remember the function of Normal probability



          $$N(mu,sigma)=displaystyle{int_{-infty}^{+infty}{frac{1}{sigmasqrt{2pi}}e^{-frac{1}{2}left(frac{x-mu}{sigma}right)^2};dx}=bf{1}}$$



          For $mu=0$ and $sigma=1$
          $$displaystyle{int_{-infty}^{+infty}{frac{1}{sqrt{2pi}}e^{-frac{x^2}{2}};dx}=bf{1}}$$



          Introduce a change of variable with its respective Jacobian
          $$x=sqrt{2};u\dx=sqrt{2};du$$



          and result
          $$displaystyle{int_{-infty}^{+infty}{frac{sqrt{2}}{sqrt{2pi}}e^{-u^2};du}=bf{1}} Rightarrow displaystyle{int_{-infty}^{+infty}{e^{-u^2};du}=bf{sqrt{pi}}}$$



          The Normal is even function, therefore
          $$displaystyle{int_0^{+infty}{e^{-u^2};du}=frac{sqrt{pi}}{2}}$$



          P.D.: Excuse my English, please.






          share|cite|improve this answer









          $endgroup$









          • 3




            $begingroup$
            I didn't downvote, but I comment in order to explain that vote: you assume that a more complicated integral is defined in order to prove that $int e^{-x^2} , dx$ is defined. Of course, integrability of the two functions $e^{-x^2}$ and the Gaussian is equivalent by a simple substitution (part of that statement is what you showed). That's begging the question
            $endgroup$
            – t.b.
            Mar 18 '12 at 23:50












          Your Answer





          StackExchange.ifUsing("editor", function () {
          return StackExchange.using("mathjaxEditing", function () {
          StackExchange.MarkdownEditor.creationCallbacks.add(function (editor, postfix) {
          StackExchange.mathjaxEditing.prepareWmdForMathJax(editor, postfix, [["$", "$"], ["\\(","\\)"]]);
          });
          });
          }, "mathjax-editing");

          StackExchange.ready(function() {
          var channelOptions = {
          tags: "".split(" "),
          id: "69"
          };
          initTagRenderer("".split(" "), "".split(" "), channelOptions);

          StackExchange.using("externalEditor", function() {
          // Have to fire editor after snippets, if snippets enabled
          if (StackExchange.settings.snippets.snippetsEnabled) {
          StackExchange.using("snippets", function() {
          createEditor();
          });
          }
          else {
          createEditor();
          }
          });

          function createEditor() {
          StackExchange.prepareEditor({
          heartbeatType: 'answer',
          autoActivateHeartbeat: false,
          convertImagesToLinks: true,
          noModals: true,
          showLowRepImageUploadWarning: true,
          reputationToPostImages: 10,
          bindNavPrevention: true,
          postfix: "",
          imageUploader: {
          brandingHtml: "Powered by u003ca class="icon-imgur-white" href="https://imgur.com/"u003eu003c/au003e",
          contentPolicyHtml: "User contributions licensed under u003ca href="https://creativecommons.org/licenses/by-sa/3.0/"u003ecc by-sa 3.0 with attribution requiredu003c/au003e u003ca href="https://stackoverflow.com/legal/content-policy"u003e(content policy)u003c/au003e",
          allowUrls: true
          },
          noCode: true, onDemand: true,
          discardSelector: ".discard-answer"
          ,immediatelyShowMarkdownHelp:true
          });


          }
          });














          draft saved

          draft discarded


















          StackExchange.ready(
          function () {
          StackExchange.openid.initPostLogin('.new-post-login', 'https%3a%2f%2fmath.stackexchange.com%2fquestions%2f96840%2fdoes-the-improper-integral-int-0-infty-e-x2dx-converge%23new-answer', 'question_page');
          }
          );

          Post as a guest















          Required, but never shown

























          6 Answers
          6






          active

          oldest

          votes








          6 Answers
          6






          active

          oldest

          votes









          active

          oldest

          votes






          active

          oldest

          votes









          17












          $begingroup$

          Write



          $$int_0^infty e^{-x^2} , dx = int_0^1 e^{-x^2} , dx + int_1^infty e^{-x^2} , dx$$



          The first integral on the right hand side is of a bounded function over a bounded interval, hence finite. For the second notice that we have $x^2 > x$ when $x > 1$. Therefore, $e^{-x^2} < e^{-x}$ for $x > 1$ and it follows that



          $$int_1^infty e^{-x^2} , dx < int_1^infty e^{-x} , dx$$
          $$= lim_{x to infty} -e^{-x} + e^{-1} = 1/e < infty$$






          share|cite|improve this answer











          $endgroup$


















            17












            $begingroup$

            Write



            $$int_0^infty e^{-x^2} , dx = int_0^1 e^{-x^2} , dx + int_1^infty e^{-x^2} , dx$$



            The first integral on the right hand side is of a bounded function over a bounded interval, hence finite. For the second notice that we have $x^2 > x$ when $x > 1$. Therefore, $e^{-x^2} < e^{-x}$ for $x > 1$ and it follows that



            $$int_1^infty e^{-x^2} , dx < int_1^infty e^{-x} , dx$$
            $$= lim_{x to infty} -e^{-x} + e^{-1} = 1/e < infty$$






            share|cite|improve this answer











            $endgroup$
















              17












              17








              17





              $begingroup$

              Write



              $$int_0^infty e^{-x^2} , dx = int_0^1 e^{-x^2} , dx + int_1^infty e^{-x^2} , dx$$



              The first integral on the right hand side is of a bounded function over a bounded interval, hence finite. For the second notice that we have $x^2 > x$ when $x > 1$. Therefore, $e^{-x^2} < e^{-x}$ for $x > 1$ and it follows that



              $$int_1^infty e^{-x^2} , dx < int_1^infty e^{-x} , dx$$
              $$= lim_{x to infty} -e^{-x} + e^{-1} = 1/e < infty$$






              share|cite|improve this answer











              $endgroup$



              Write



              $$int_0^infty e^{-x^2} , dx = int_0^1 e^{-x^2} , dx + int_1^infty e^{-x^2} , dx$$



              The first integral on the right hand side is of a bounded function over a bounded interval, hence finite. For the second notice that we have $x^2 > x$ when $x > 1$. Therefore, $e^{-x^2} < e^{-x}$ for $x > 1$ and it follows that



              $$int_1^infty e^{-x^2} , dx < int_1^infty e^{-x} , dx$$
              $$= lim_{x to infty} -e^{-x} + e^{-1} = 1/e < infty$$







              share|cite|improve this answer














              share|cite|improve this answer



              share|cite|improve this answer








              edited Jan 9 '12 at 20:44

























              answered Jan 6 '12 at 4:46







              user12014






























                  4












                  $begingroup$

                  I'm assuming you're asking about the convergence of $int_0^infty e^{-x^2}dx$. The easiest way that I can think of to prove this is to note that $e^{-x^2}$ is continuous and bounded, and hence integrable, on the interval $[0,1]$, and that on the remaining unbounded interval $[1,infty)$ it is a function everywhere bounded in absolute value by a function $e^{-x}$ that is integrable on that interval (seeing that $int_1^infty e^{-x}dx$ converges is a simple calculation, since $e^{-x}$ has an easy antiderivative). Thus, by the comparison test, $int_0^infty e^{-x^2}dx$ converges. I think this is the idea you're referring to in your question.



                  $int_0^infty e^{-x^2}dx$ does not exist if $int_0^infty dx$ does, since the latter one most certainly does not converge (it equals $lim_{x to infty} x = infty$). While it is true that $e^{-x^2} le 1$ on $[0,infty)$, this fact isn't really helpful, because the comparison test only gives us information when the bounding function is itself integrable (otherwise you get absurdities like claiming that $int_0^infty e^{-x}dx$ diverges because $e^{-x} le x $ and $int_0^infty x dx$ diverges.)






                  share|cite|improve this answer









                  $endgroup$


















                    4












                    $begingroup$

                    I'm assuming you're asking about the convergence of $int_0^infty e^{-x^2}dx$. The easiest way that I can think of to prove this is to note that $e^{-x^2}$ is continuous and bounded, and hence integrable, on the interval $[0,1]$, and that on the remaining unbounded interval $[1,infty)$ it is a function everywhere bounded in absolute value by a function $e^{-x}$ that is integrable on that interval (seeing that $int_1^infty e^{-x}dx$ converges is a simple calculation, since $e^{-x}$ has an easy antiderivative). Thus, by the comparison test, $int_0^infty e^{-x^2}dx$ converges. I think this is the idea you're referring to in your question.



                    $int_0^infty e^{-x^2}dx$ does not exist if $int_0^infty dx$ does, since the latter one most certainly does not converge (it equals $lim_{x to infty} x = infty$). While it is true that $e^{-x^2} le 1$ on $[0,infty)$, this fact isn't really helpful, because the comparison test only gives us information when the bounding function is itself integrable (otherwise you get absurdities like claiming that $int_0^infty e^{-x}dx$ diverges because $e^{-x} le x $ and $int_0^infty x dx$ diverges.)






                    share|cite|improve this answer









                    $endgroup$
















                      4












                      4








                      4





                      $begingroup$

                      I'm assuming you're asking about the convergence of $int_0^infty e^{-x^2}dx$. The easiest way that I can think of to prove this is to note that $e^{-x^2}$ is continuous and bounded, and hence integrable, on the interval $[0,1]$, and that on the remaining unbounded interval $[1,infty)$ it is a function everywhere bounded in absolute value by a function $e^{-x}$ that is integrable on that interval (seeing that $int_1^infty e^{-x}dx$ converges is a simple calculation, since $e^{-x}$ has an easy antiderivative). Thus, by the comparison test, $int_0^infty e^{-x^2}dx$ converges. I think this is the idea you're referring to in your question.



                      $int_0^infty e^{-x^2}dx$ does not exist if $int_0^infty dx$ does, since the latter one most certainly does not converge (it equals $lim_{x to infty} x = infty$). While it is true that $e^{-x^2} le 1$ on $[0,infty)$, this fact isn't really helpful, because the comparison test only gives us information when the bounding function is itself integrable (otherwise you get absurdities like claiming that $int_0^infty e^{-x}dx$ diverges because $e^{-x} le x $ and $int_0^infty x dx$ diverges.)






                      share|cite|improve this answer









                      $endgroup$



                      I'm assuming you're asking about the convergence of $int_0^infty e^{-x^2}dx$. The easiest way that I can think of to prove this is to note that $e^{-x^2}$ is continuous and bounded, and hence integrable, on the interval $[0,1]$, and that on the remaining unbounded interval $[1,infty)$ it is a function everywhere bounded in absolute value by a function $e^{-x}$ that is integrable on that interval (seeing that $int_1^infty e^{-x}dx$ converges is a simple calculation, since $e^{-x}$ has an easy antiderivative). Thus, by the comparison test, $int_0^infty e^{-x^2}dx$ converges. I think this is the idea you're referring to in your question.



                      $int_0^infty e^{-x^2}dx$ does not exist if $int_0^infty dx$ does, since the latter one most certainly does not converge (it equals $lim_{x to infty} x = infty$). While it is true that $e^{-x^2} le 1$ on $[0,infty)$, this fact isn't really helpful, because the comparison test only gives us information when the bounding function is itself integrable (otherwise you get absurdities like claiming that $int_0^infty e^{-x}dx$ diverges because $e^{-x} le x $ and $int_0^infty x dx$ diverges.)







                      share|cite|improve this answer












                      share|cite|improve this answer



                      share|cite|improve this answer










                      answered Jan 6 '12 at 4:47









                      Calvin McPhail-SnyderCalvin McPhail-Snyder

                      1,37411120




                      1,37411120























                          3












                          $begingroup$

                          It does, and you can also compute its value:



                          $int_{[0, infty) times[0, infty)} e^{-(x^2+y^2)} dx dy = int_{[0, infty)}big(int_{[0, infty)} e^{-(x^2+y^2)} dy big)dx = int_{[0, infty)} e ^ {-x^2}big(int_{[0, infty)} e^{-y^2} dy big)dx =$
                          $= int_{[0, infty)} e ^ {-x^2}dx int_{[0, infty)} e^{-y^2} dy = big( int_{[0, infty)} e ^ {-x^2}dx big)^2$



                          Then we can use polar coordinates:



                          $int_{[0, infty) times[0, infty)} e^{-(x^2+y^2)} dx dy = int_0^{infty} big(int_0^{fracpi2}re^{-r^2}dtheta)dr = fracpi2int_0^{infty}re^{-r^2}dr = fracpi2 frac12 = fracpi4$



                          Therefore:



                          $int_{[0, infty)} e ^ {-x^2}dx = big(fracpi4big)^{frac12}$






                          share|cite|improve this answer











                          $endgroup$


















                            3












                            $begingroup$

                            It does, and you can also compute its value:



                            $int_{[0, infty) times[0, infty)} e^{-(x^2+y^2)} dx dy = int_{[0, infty)}big(int_{[0, infty)} e^{-(x^2+y^2)} dy big)dx = int_{[0, infty)} e ^ {-x^2}big(int_{[0, infty)} e^{-y^2} dy big)dx =$
                            $= int_{[0, infty)} e ^ {-x^2}dx int_{[0, infty)} e^{-y^2} dy = big( int_{[0, infty)} e ^ {-x^2}dx big)^2$



                            Then we can use polar coordinates:



                            $int_{[0, infty) times[0, infty)} e^{-(x^2+y^2)} dx dy = int_0^{infty} big(int_0^{fracpi2}re^{-r^2}dtheta)dr = fracpi2int_0^{infty}re^{-r^2}dr = fracpi2 frac12 = fracpi4$



                            Therefore:



                            $int_{[0, infty)} e ^ {-x^2}dx = big(fracpi4big)^{frac12}$






                            share|cite|improve this answer











                            $endgroup$
















                              3












                              3








                              3





                              $begingroup$

                              It does, and you can also compute its value:



                              $int_{[0, infty) times[0, infty)} e^{-(x^2+y^2)} dx dy = int_{[0, infty)}big(int_{[0, infty)} e^{-(x^2+y^2)} dy big)dx = int_{[0, infty)} e ^ {-x^2}big(int_{[0, infty)} e^{-y^2} dy big)dx =$
                              $= int_{[0, infty)} e ^ {-x^2}dx int_{[0, infty)} e^{-y^2} dy = big( int_{[0, infty)} e ^ {-x^2}dx big)^2$



                              Then we can use polar coordinates:



                              $int_{[0, infty) times[0, infty)} e^{-(x^2+y^2)} dx dy = int_0^{infty} big(int_0^{fracpi2}re^{-r^2}dtheta)dr = fracpi2int_0^{infty}re^{-r^2}dr = fracpi2 frac12 = fracpi4$



                              Therefore:



                              $int_{[0, infty)} e ^ {-x^2}dx = big(fracpi4big)^{frac12}$






                              share|cite|improve this answer











                              $endgroup$



                              It does, and you can also compute its value:



                              $int_{[0, infty) times[0, infty)} e^{-(x^2+y^2)} dx dy = int_{[0, infty)}big(int_{[0, infty)} e^{-(x^2+y^2)} dy big)dx = int_{[0, infty)} e ^ {-x^2}big(int_{[0, infty)} e^{-y^2} dy big)dx =$
                              $= int_{[0, infty)} e ^ {-x^2}dx int_{[0, infty)} e^{-y^2} dy = big( int_{[0, infty)} e ^ {-x^2}dx big)^2$



                              Then we can use polar coordinates:



                              $int_{[0, infty) times[0, infty)} e^{-(x^2+y^2)} dx dy = int_0^{infty} big(int_0^{fracpi2}re^{-r^2}dtheta)dr = fracpi2int_0^{infty}re^{-r^2}dr = fracpi2 frac12 = fracpi4$



                              Therefore:



                              $int_{[0, infty)} e ^ {-x^2}dx = big(fracpi4big)^{frac12}$







                              share|cite|improve this answer














                              share|cite|improve this answer



                              share|cite|improve this answer








                              edited Jan 7 '12 at 6:16









                              Jonas Meyer

                              41.1k6149259




                              41.1k6149259










                              answered Jan 6 '12 at 7:00









                              wazabitwazabit

                              20715




                              20715























                                  1












                                  $begingroup$

                                  Exercise 3-41 of Spivak's "Calculus on Manifolds" outlines how to prove that $int_{-infty}^{infty}e^{-x^{2}}dx=sqrt{pi}$ using polar coordinates, a related double integral, and two useful families of regions over which to evaluate the double integral. The description is actually quite long. Ironically, he ends the problem with a quote attributed to Lord Kelvin (William Thomson), who was trying to convey the meaning of "mathematician" to a class: "A mathematician is one to whom "that" (i.e., $int_{-infty}^{infty}e^{-x^{2}}dx=sqrt{pi}$) is as obvious as that twice two makes four is to you." A bit harsh, I daresay, on Lord Kelvin's part. The reality being conveyed by Spivak is that it's obvious once you've done the necessary work.






                                  share|cite|improve this answer









                                  $endgroup$









                                  • 2




                                    $begingroup$
                                    Does Spivak quote the next part? "Liouville was a mathematician." I always thought that was a nice thing to say about someone.
                                    $endgroup$
                                    – Dylan Moreland
                                    Jan 6 '12 at 6:54










                                  • $begingroup$
                                    @Dylan-Yes. Spivak includes that statement.
                                    $endgroup$
                                    – Chris Leary
                                    Jan 6 '12 at 18:15
















                                  1












                                  $begingroup$

                                  Exercise 3-41 of Spivak's "Calculus on Manifolds" outlines how to prove that $int_{-infty}^{infty}e^{-x^{2}}dx=sqrt{pi}$ using polar coordinates, a related double integral, and two useful families of regions over which to evaluate the double integral. The description is actually quite long. Ironically, he ends the problem with a quote attributed to Lord Kelvin (William Thomson), who was trying to convey the meaning of "mathematician" to a class: "A mathematician is one to whom "that" (i.e., $int_{-infty}^{infty}e^{-x^{2}}dx=sqrt{pi}$) is as obvious as that twice two makes four is to you." A bit harsh, I daresay, on Lord Kelvin's part. The reality being conveyed by Spivak is that it's obvious once you've done the necessary work.






                                  share|cite|improve this answer









                                  $endgroup$









                                  • 2




                                    $begingroup$
                                    Does Spivak quote the next part? "Liouville was a mathematician." I always thought that was a nice thing to say about someone.
                                    $endgroup$
                                    – Dylan Moreland
                                    Jan 6 '12 at 6:54










                                  • $begingroup$
                                    @Dylan-Yes. Spivak includes that statement.
                                    $endgroup$
                                    – Chris Leary
                                    Jan 6 '12 at 18:15














                                  1












                                  1








                                  1





                                  $begingroup$

                                  Exercise 3-41 of Spivak's "Calculus on Manifolds" outlines how to prove that $int_{-infty}^{infty}e^{-x^{2}}dx=sqrt{pi}$ using polar coordinates, a related double integral, and two useful families of regions over which to evaluate the double integral. The description is actually quite long. Ironically, he ends the problem with a quote attributed to Lord Kelvin (William Thomson), who was trying to convey the meaning of "mathematician" to a class: "A mathematician is one to whom "that" (i.e., $int_{-infty}^{infty}e^{-x^{2}}dx=sqrt{pi}$) is as obvious as that twice two makes four is to you." A bit harsh, I daresay, on Lord Kelvin's part. The reality being conveyed by Spivak is that it's obvious once you've done the necessary work.






                                  share|cite|improve this answer









                                  $endgroup$



                                  Exercise 3-41 of Spivak's "Calculus on Manifolds" outlines how to prove that $int_{-infty}^{infty}e^{-x^{2}}dx=sqrt{pi}$ using polar coordinates, a related double integral, and two useful families of regions over which to evaluate the double integral. The description is actually quite long. Ironically, he ends the problem with a quote attributed to Lord Kelvin (William Thomson), who was trying to convey the meaning of "mathematician" to a class: "A mathematician is one to whom "that" (i.e., $int_{-infty}^{infty}e^{-x^{2}}dx=sqrt{pi}$) is as obvious as that twice two makes four is to you." A bit harsh, I daresay, on Lord Kelvin's part. The reality being conveyed by Spivak is that it's obvious once you've done the necessary work.







                                  share|cite|improve this answer












                                  share|cite|improve this answer



                                  share|cite|improve this answer










                                  answered Jan 6 '12 at 6:44









                                  Chris LearyChris Leary

                                  1,8901316




                                  1,8901316








                                  • 2




                                    $begingroup$
                                    Does Spivak quote the next part? "Liouville was a mathematician." I always thought that was a nice thing to say about someone.
                                    $endgroup$
                                    – Dylan Moreland
                                    Jan 6 '12 at 6:54










                                  • $begingroup$
                                    @Dylan-Yes. Spivak includes that statement.
                                    $endgroup$
                                    – Chris Leary
                                    Jan 6 '12 at 18:15














                                  • 2




                                    $begingroup$
                                    Does Spivak quote the next part? "Liouville was a mathematician." I always thought that was a nice thing to say about someone.
                                    $endgroup$
                                    – Dylan Moreland
                                    Jan 6 '12 at 6:54










                                  • $begingroup$
                                    @Dylan-Yes. Spivak includes that statement.
                                    $endgroup$
                                    – Chris Leary
                                    Jan 6 '12 at 18:15








                                  2




                                  2




                                  $begingroup$
                                  Does Spivak quote the next part? "Liouville was a mathematician." I always thought that was a nice thing to say about someone.
                                  $endgroup$
                                  – Dylan Moreland
                                  Jan 6 '12 at 6:54




                                  $begingroup$
                                  Does Spivak quote the next part? "Liouville was a mathematician." I always thought that was a nice thing to say about someone.
                                  $endgroup$
                                  – Dylan Moreland
                                  Jan 6 '12 at 6:54












                                  $begingroup$
                                  @Dylan-Yes. Spivak includes that statement.
                                  $endgroup$
                                  – Chris Leary
                                  Jan 6 '12 at 18:15




                                  $begingroup$
                                  @Dylan-Yes. Spivak includes that statement.
                                  $endgroup$
                                  – Chris Leary
                                  Jan 6 '12 at 18:15











                                  0












                                  $begingroup$

                                  Let $$f(x) = frac{1}{{e^x}^{2}}$$
                                  Now let $$g(x) = frac{1}{x^2 + 1}$$
                                  $$lim _{xrightarrow infty} = 0 $$ and $g(x)$ is a convergent integral, so $f(x)$ is convergent too.






                                  share|cite|improve this answer











                                  $endgroup$


















                                    0












                                    $begingroup$

                                    Let $$f(x) = frac{1}{{e^x}^{2}}$$
                                    Now let $$g(x) = frac{1}{x^2 + 1}$$
                                    $$lim _{xrightarrow infty} = 0 $$ and $g(x)$ is a convergent integral, so $f(x)$ is convergent too.






                                    share|cite|improve this answer











                                    $endgroup$
















                                      0












                                      0








                                      0





                                      $begingroup$

                                      Let $$f(x) = frac{1}{{e^x}^{2}}$$
                                      Now let $$g(x) = frac{1}{x^2 + 1}$$
                                      $$lim _{xrightarrow infty} = 0 $$ and $g(x)$ is a convergent integral, so $f(x)$ is convergent too.






                                      share|cite|improve this answer











                                      $endgroup$



                                      Let $$f(x) = frac{1}{{e^x}^{2}}$$
                                      Now let $$g(x) = frac{1}{x^2 + 1}$$
                                      $$lim _{xrightarrow infty} = 0 $$ and $g(x)$ is a convergent integral, so $f(x)$ is convergent too.







                                      share|cite|improve this answer














                                      share|cite|improve this answer



                                      share|cite|improve this answer








                                      edited Dec 20 '18 at 18:38









                                      Prakhar Nagpal

                                      754318




                                      754318










                                      answered Dec 20 '18 at 18:08









                                      Ali nakhaeeAli nakhaee

                                      1




                                      1























                                          -3












                                          $begingroup$

                                          A simple answer as @neemy… Remember the function of Normal probability



                                          $$N(mu,sigma)=displaystyle{int_{-infty}^{+infty}{frac{1}{sigmasqrt{2pi}}e^{-frac{1}{2}left(frac{x-mu}{sigma}right)^2};dx}=bf{1}}$$



                                          For $mu=0$ and $sigma=1$
                                          $$displaystyle{int_{-infty}^{+infty}{frac{1}{sqrt{2pi}}e^{-frac{x^2}{2}};dx}=bf{1}}$$



                                          Introduce a change of variable with its respective Jacobian
                                          $$x=sqrt{2};u\dx=sqrt{2};du$$



                                          and result
                                          $$displaystyle{int_{-infty}^{+infty}{frac{sqrt{2}}{sqrt{2pi}}e^{-u^2};du}=bf{1}} Rightarrow displaystyle{int_{-infty}^{+infty}{e^{-u^2};du}=bf{sqrt{pi}}}$$



                                          The Normal is even function, therefore
                                          $$displaystyle{int_0^{+infty}{e^{-u^2};du}=frac{sqrt{pi}}{2}}$$



                                          P.D.: Excuse my English, please.






                                          share|cite|improve this answer









                                          $endgroup$









                                          • 3




                                            $begingroup$
                                            I didn't downvote, but I comment in order to explain that vote: you assume that a more complicated integral is defined in order to prove that $int e^{-x^2} , dx$ is defined. Of course, integrability of the two functions $e^{-x^2}$ and the Gaussian is equivalent by a simple substitution (part of that statement is what you showed). That's begging the question
                                            $endgroup$
                                            – t.b.
                                            Mar 18 '12 at 23:50
















                                          -3












                                          $begingroup$

                                          A simple answer as @neemy… Remember the function of Normal probability



                                          $$N(mu,sigma)=displaystyle{int_{-infty}^{+infty}{frac{1}{sigmasqrt{2pi}}e^{-frac{1}{2}left(frac{x-mu}{sigma}right)^2};dx}=bf{1}}$$



                                          For $mu=0$ and $sigma=1$
                                          $$displaystyle{int_{-infty}^{+infty}{frac{1}{sqrt{2pi}}e^{-frac{x^2}{2}};dx}=bf{1}}$$



                                          Introduce a change of variable with its respective Jacobian
                                          $$x=sqrt{2};u\dx=sqrt{2};du$$



                                          and result
                                          $$displaystyle{int_{-infty}^{+infty}{frac{sqrt{2}}{sqrt{2pi}}e^{-u^2};du}=bf{1}} Rightarrow displaystyle{int_{-infty}^{+infty}{e^{-u^2};du}=bf{sqrt{pi}}}$$



                                          The Normal is even function, therefore
                                          $$displaystyle{int_0^{+infty}{e^{-u^2};du}=frac{sqrt{pi}}{2}}$$



                                          P.D.: Excuse my English, please.






                                          share|cite|improve this answer









                                          $endgroup$









                                          • 3




                                            $begingroup$
                                            I didn't downvote, but I comment in order to explain that vote: you assume that a more complicated integral is defined in order to prove that $int e^{-x^2} , dx$ is defined. Of course, integrability of the two functions $e^{-x^2}$ and the Gaussian is equivalent by a simple substitution (part of that statement is what you showed). That's begging the question
                                            $endgroup$
                                            – t.b.
                                            Mar 18 '12 at 23:50














                                          -3












                                          -3








                                          -3





                                          $begingroup$

                                          A simple answer as @neemy… Remember the function of Normal probability



                                          $$N(mu,sigma)=displaystyle{int_{-infty}^{+infty}{frac{1}{sigmasqrt{2pi}}e^{-frac{1}{2}left(frac{x-mu}{sigma}right)^2};dx}=bf{1}}$$



                                          For $mu=0$ and $sigma=1$
                                          $$displaystyle{int_{-infty}^{+infty}{frac{1}{sqrt{2pi}}e^{-frac{x^2}{2}};dx}=bf{1}}$$



                                          Introduce a change of variable with its respective Jacobian
                                          $$x=sqrt{2};u\dx=sqrt{2};du$$



                                          and result
                                          $$displaystyle{int_{-infty}^{+infty}{frac{sqrt{2}}{sqrt{2pi}}e^{-u^2};du}=bf{1}} Rightarrow displaystyle{int_{-infty}^{+infty}{e^{-u^2};du}=bf{sqrt{pi}}}$$



                                          The Normal is even function, therefore
                                          $$displaystyle{int_0^{+infty}{e^{-u^2};du}=frac{sqrt{pi}}{2}}$$



                                          P.D.: Excuse my English, please.






                                          share|cite|improve this answer









                                          $endgroup$



                                          A simple answer as @neemy… Remember the function of Normal probability



                                          $$N(mu,sigma)=displaystyle{int_{-infty}^{+infty}{frac{1}{sigmasqrt{2pi}}e^{-frac{1}{2}left(frac{x-mu}{sigma}right)^2};dx}=bf{1}}$$



                                          For $mu=0$ and $sigma=1$
                                          $$displaystyle{int_{-infty}^{+infty}{frac{1}{sqrt{2pi}}e^{-frac{x^2}{2}};dx}=bf{1}}$$



                                          Introduce a change of variable with its respective Jacobian
                                          $$x=sqrt{2};u\dx=sqrt{2};du$$



                                          and result
                                          $$displaystyle{int_{-infty}^{+infty}{frac{sqrt{2}}{sqrt{2pi}}e^{-u^2};du}=bf{1}} Rightarrow displaystyle{int_{-infty}^{+infty}{e^{-u^2};du}=bf{sqrt{pi}}}$$



                                          The Normal is even function, therefore
                                          $$displaystyle{int_0^{+infty}{e^{-u^2};du}=frac{sqrt{pi}}{2}}$$



                                          P.D.: Excuse my English, please.







                                          share|cite|improve this answer












                                          share|cite|improve this answer



                                          share|cite|improve this answer










                                          answered Mar 18 '12 at 19:56









                                          diofantodiofanto

                                          1142




                                          1142








                                          • 3




                                            $begingroup$
                                            I didn't downvote, but I comment in order to explain that vote: you assume that a more complicated integral is defined in order to prove that $int e^{-x^2} , dx$ is defined. Of course, integrability of the two functions $e^{-x^2}$ and the Gaussian is equivalent by a simple substitution (part of that statement is what you showed). That's begging the question
                                            $endgroup$
                                            – t.b.
                                            Mar 18 '12 at 23:50














                                          • 3




                                            $begingroup$
                                            I didn't downvote, but I comment in order to explain that vote: you assume that a more complicated integral is defined in order to prove that $int e^{-x^2} , dx$ is defined. Of course, integrability of the two functions $e^{-x^2}$ and the Gaussian is equivalent by a simple substitution (part of that statement is what you showed). That's begging the question
                                            $endgroup$
                                            – t.b.
                                            Mar 18 '12 at 23:50








                                          3




                                          3




                                          $begingroup$
                                          I didn't downvote, but I comment in order to explain that vote: you assume that a more complicated integral is defined in order to prove that $int e^{-x^2} , dx$ is defined. Of course, integrability of the two functions $e^{-x^2}$ and the Gaussian is equivalent by a simple substitution (part of that statement is what you showed). That's begging the question
                                          $endgroup$
                                          – t.b.
                                          Mar 18 '12 at 23:50




                                          $begingroup$
                                          I didn't downvote, but I comment in order to explain that vote: you assume that a more complicated integral is defined in order to prove that $int e^{-x^2} , dx$ is defined. Of course, integrability of the two functions $e^{-x^2}$ and the Gaussian is equivalent by a simple substitution (part of that statement is what you showed). That's begging the question
                                          $endgroup$
                                          – t.b.
                                          Mar 18 '12 at 23:50


















                                          draft saved

                                          draft discarded




















































                                          Thanks for contributing an answer to Mathematics Stack Exchange!


                                          • Please be sure to answer the question. Provide details and share your research!

                                          But avoid



                                          • Asking for help, clarification, or responding to other answers.

                                          • Making statements based on opinion; back them up with references or personal experience.


                                          Use MathJax to format equations. MathJax reference.


                                          To learn more, see our tips on writing great answers.




                                          draft saved


                                          draft discarded














                                          StackExchange.ready(
                                          function () {
                                          StackExchange.openid.initPostLogin('.new-post-login', 'https%3a%2f%2fmath.stackexchange.com%2fquestions%2f96840%2fdoes-the-improper-integral-int-0-infty-e-x2dx-converge%23new-answer', 'question_page');
                                          }
                                          );

                                          Post as a guest















                                          Required, but never shown





















































                                          Required, but never shown














                                          Required, but never shown












                                          Required, but never shown







                                          Required, but never shown

































                                          Required, but never shown














                                          Required, but never shown












                                          Required, but never shown







                                          Required, but never shown







                                          Popular posts from this blog

                                          Plaza Victoria

                                          In PowerPoint, is there a keyboard shortcut for bulleted / numbered list?

                                          How to put 3 figures in Latex with 2 figures side by side and 1 below these side by side images but in...